\documentclass[a5paper,10pt]{article} \usepackage{myXsim} \usepackage{tasks} % Title Page \title{DM2 \hfill BENHATTAL Chakir} \tribe{TST} \date{\hfillÀ render pour le Mercredi 24 février} \xsimsetup{ solution/print = false } \begin{document} \maketitle \begin{exercise}[subtitle={Loi binomiale}] Trois personnes s'apprêtent à passer le portique de sécurité. On suppose que pour chaque personne la probabilité que le portique sonne est égale à $0.16$. Soit $X$ la variable aléatoire donnant le nombre de personnes faisant sonner le portique, parmi les 3 personnes de ce groupe. \begin{enumerate} \item Tracer l'arbre représentant le situation. \item Justifier que $X$ suit une loi binomiale dont on précisera les paramètres. \item Quelle est la probabilité qu'une seule personne fasse sonner le portique? \item Calculer puis interpréter les probabilités suivantes \[ P(X = 0) \qquad \qquad P(X \geq 2) \] \item Calculer l'espérance de $X$ et interpréter le résultat. \end{enumerate} \end{exercise} \begin{solution} \begin{enumerate} \item \begin{tikzpicture}[sloped] \node {.} child {node {$0$} child {node {$0$} child {node {$0$} edge from parent node[above] {0.84} } child {node {$1$} edge from parent node[above] {0.16} } edge from parent node[above] {0.84} } child[missing] {} child {node {$1$} child {node {$0$} edge from parent node[above] {0.84} } child {node {$1$} edge from parent node[above] {0.16} } edge from parent node[above] {0.84} } edge from parent node[above] {0.84} } child[missing] {} child[missing] {} child[missing] {} child { node {$1$} child {node {$0$} child {node {$0$} edge from parent node[above] {0.84} } child {node {$1$} edge from parent node[above] {0.16} } edge from parent node[above] {0.84} } child[missing] {} child {node {$1$} child {node {$0$} edge from parent node[above] {0.84} } child {node {$1$} edge from parent node[above] {0.16} } edge from parent node[above] {0.84} } edge from parent node[above] {0.16} } ; \end{tikzpicture} \item Chaque personne a 2 possibilités (1: fait sonner ou 2: ne fait pas sonner) et l'on fait passer 3 personnes ce qui correspond à une répétition identique et aléatoire. On peut donc modéliser la situation par une loi binomiale. \[ X \sim \mathcal{B}(3; 0.76) \] \item Probabilité qu'une seule personne fasse sonner le portique. On voit qu'il y a 3 branches qui correspondent à cette situation dont \[ P(X = 1) = 3 \times 0.16^1 \times 0.84^2 \approx 0.339 \] \item \[ P(X = 0) = 0.84^3 \approx 0.593 \] \[ P(X \geq 2) = P(X = 2) + P(X = 3) = 3 \times 0.16^2 \times 0.84^1 + 0.16^3 \approx 0.069 \] \item Il faut d'abord tracer le tableau résumant la loi de probabilité: \begin{center} \begin{tabular}{|c|*{4}{c|}} \hline Valeur & 0 & 1 & 2 & 3 \\ \hline Probabilité & $0.593$ & $0.339$ & $0.065$ &$0.004$ \\ \hline \end{tabular} \end{center} On peut alors calculer l'espérance \[ E[X] = 0 \times 0.593 + 1 \times 0.339 + 2 \times 0.065 + 3 \times 0.004 = 0.48 \] On peut donc estimer qu'il y aura en moyenne $0.48$ personnes qui feront sonner le portique sur les 3 personnes. \end{enumerate} \end{solution} \begin{exercise}[subtitle={Équation puissance}] Résoudre les équations et inéquations suivantes \begin{multicols}{2} \begin{enumerate} \item $10^x = 31$ \item $11^x = 39$ \item $0.65^x \leq 46$ \item $2 \times 0.75^x = 11$ \end{enumerate} \end{multicols} \end{exercise} \begin{solution} Les solutions ci-dessous ne sont pas justifiée car l'ordinateur ne sait pas faire. Par contre, vous vous devez savoir justifier vos réponses! \begin{enumerate} \item $x = \log(31)$ \item $x = \frac{\log(39)}{\log(11)}$ \item Il faut faire attention quand on divise par un log car ce dernier peut être négatif ce qui est le cas ici. Il faut donc pense à changer le sens de l'inégalité. $x \geq \frac{\log(46)}{\log(0.65)}$ \item Il faut penser à faire la division à par $2$ avant d'utiliser le log car sinon, on ne peut pas utiliser la formule $\log(a^n) = n\times \log(a)$. $x = \frac{\log(5.5)}{\log(0.75)}$ \end{enumerate} \end{solution} \begin{exercise}[subtitle={Étude de fonctions}] Soit $f(x) = - 4x^3 + 354x^2 - 9120x + 5$ une fonction définie sur $\R$. \begin{enumerate} \item Calculer $f'(x)$ la dérivée de $f(x)$. \item Calculer $f'(40)$ et $f'(19)$. \item En déduire une forme factorisée de $f'(x)$. \item Étudier le signe de $f'(x)$ et en déduire les variations de $f(x)$. \item Est-ce que la fonction $f(x)$ admet un maximum ou un minimum? Si oui, calculer sa valeur. \end{enumerate} \end{exercise} \begin{solution} \begin{enumerate} \item Dérivée de $f(x)$: $f'(x) = - 12x^2 + 708x - 9120$ \item \begin{align*} f'(40) &= - 12 \times 40^{2} + 708 \times 40 - 9120\\&= - 12 \times 1600 + 28320 - 9120\\&= - 19200 + 19200\\&= 0 \end{align*} \begin{align*} f'(19) &= - 12 \times 19^{2} + 708 \times 19 - 9120\\&= - 12 \times 361 + 13452 - 9120\\&= - 4332 + 4332\\&= 0 \end{align*} Donc $x = 40$ et $x=19$ sont des racines de $f'(x) = - 12x^2 + 708x - 9120$. \item On en déduit la forme factorisée suivante \[ f'(x) = -12 (x - 40)(x-19) \] \item Pas de correction disponible \item À causes des branches extérieurs, la fonction $f(x)$ n'a pas de maximum ou de minimum. \end{enumerate} \end{solution} %\printsolutionstype{exercise} \end{document} %%% Local Variables: %%% mode: latex %%% TeX-master: "master" %%% End: